Why does (-1)^n(sin(pi/n)) converge when (sin(p/n)) diverges

Click For Summary
The discussion centers on the convergence of the series ∑(-1)^n*sin(π/n) despite the divergence of ∑sin(π/n). While the latter diverges due to comparison with π/n, the alternating series converges due to the properties of alternating series. The convergence is explained by the fact that the terms decrease in magnitude and alternate in sign, which allows for cancellation effects. The discussion highlights that the divergence of individual series does not imply the divergence of their alternating counterpart. This illustrates the distinct behavior of alternating series in mathematical analysis.
solour
Messages
9
Reaction score
0

Homework Statement


I know that ∑n=1 to infinity (sin(p/n)) diverges due using comparison test with pi/n, despite it approaching 0 as n approaches infinity.

However, an alternating series with (-1)^n*sin(pi/n) converges. Which does not make sense because it consists of two diverging functions.

Is there any intuitive explanation for this? Or is it just a rule that I need to remember when treating alternating series

Thank you.
 
Physics news on Phys.org
How about this: For large n, sin(π/n) is approximately π/n, so the series is approximately πΣ(1/π) which is known to diverge.

With alternating signs, the series approximates πΣ( (1/n) - 1/(n+1) ) which approaches π.ln(2). Actually I think I find it more persuasive to group pairs of successive terms:
(1/n) - 1/(n+1) = 1/(n(n+1)) = O(1/n2), but I gather that's not always a safe thing to do.
 
solour said:
However, an alternating series with (-1)^n*sin(pi/n) converges. Which does not make sense because it consists of two diverging functions.
The fact that ##\sum (-1)^n## and##\sum \sin(\pi/n)## both diverge says absolutely nothing about the alternating series. If you don't believe that, you might consider ##a_n = b_n = 1/n##. Neither ##\sum a_n## nor ##\sum b_n## converge, but ##\sum a_nb_n = \sum 1/n^2## converges.
 
  • Like
Likes Jamison Lahman
Question: A clock's minute hand has length 4 and its hour hand has length 3. What is the distance between the tips at the moment when it is increasing most rapidly?(Putnam Exam Question) Answer: Making assumption that both the hands moves at constant angular velocities, the answer is ## \sqrt{7} .## But don't you think this assumption is somewhat doubtful and wrong?

Similar threads

  • · Replies 3 ·
Replies
3
Views
1K
  • · Replies 7 ·
Replies
7
Views
2K
  • · Replies 1 ·
Replies
1
Views
1K
Replies
14
Views
2K
  • · Replies 15 ·
Replies
15
Views
2K
Replies
29
Views
5K
  • · Replies 4 ·
Replies
4
Views
1K
  • · Replies 4 ·
Replies
4
Views
20K
  • · Replies 1 ·
Replies
1
Views
2K
  • · Replies 14 ·
Replies
14
Views
2K